LSAT and Law School Admissions Forum

Get expert LSAT preparation and law school admissions advice from PowerScore Test Preparation.

User avatar
 Overthinker99
  • Posts: 14
  • Joined: May 30, 2023
|
#105794
Here’s another take on the reasoning behind this question…aside from the crucial “part to whole” assumption/flaw. The stims premise says that “because there is less demand, the junkyards do not buy it.” All that provides us is the necessary condition of our general statement, which is that “harder to sell→ less demand (contrapositive of ” that more demand→easier to sell”). Satisfying our necessary is not enough to tell us the sufficient is true–that the car will be harder to sell-- which is our conclusion.

In that sense what we need is both a sufficient and necessary assumption: that “less demand” →”harder to sell.” That is exactly what “E” gives us: demand is a sufficient indicator of salability.

D is incorrect not only because it does not bridge the gap, but because its negation would merely be that "lack of demand would be offset by lack of supply." This would strengthen the assumption more than weaken it, as we’d be left with a net-zero difference while we know that newer cars are easier to sell.

Get the most out of your LSAT Prep Plus subscription.

Analyze and track your performance with our Testing and Analytics Package.